I need help with this quadratic function… I thought I knew the answer, but obviously I don’t

I Need Help With This Quadratic Function I Thought I Knew The Answer, But Obviously I Dont

Answers

Answer 1

Let us start with the following quadratic function:

[tex]f(x)=x^2-x-12[/tex]

the X-intercepts are the collection of values to X which makes f(x) = 0, and it can be calculated by the Bhaskara formula:

[tex]x_{1,2}=\frac{-b\pm\sqrt{b^2-4ac}}{2a}[/tex]

where the values a, b, and c are given by:

[tex]f(x)=ax^2+bx+c[/tex]

Substituting the values from the proposed equation, we have:

[tex]\begin{gathered} x_{1,2}=\frac{1\pm\sqrt{1^2-4*1*(-12)}}{2*1} \\ x_{1,2}=\frac{1\pm\sqrt{1+48}}{2}=\frac{1\pm\sqrt{49}}{2} \\ x_{1,2}=\frac{1\pm7}{2} \\ \\ x_1=\frac{1+7}{2}=\frac{8}{2}=4 \\ x_2=\frac{1-7}{2}=-\frac{6}{2}=-3 \end{gathered}[/tex]

From the above-developed solution, we are able to conclude that the solution for the first box is:

(-3,0) ,(4,0)

Now, the y-intercept, is just the value of y when x = 0, which can be calculated as follows:

[tex]\begin{gathered} f(0)=0^2-0-12=-12 \\ f(0)=-12 \end{gathered}[/tex]

From this, we are able to conclude that the solution for the second box is:

(0, -12)

Now, the vertex is the value of minimum, or maximum, in the quadratic equation, and use to be calculated as follows:

[tex]\begin{gathered} Vertex \\ x=-\frac{b}{2a} \\ y=\frac{4ac-b^2}{2a} \end{gathered}[/tex]

substituting the values, we have:

[tex]\begin{gathered} x=-\frac{-1}{2*1}=\frac{1}{2} \\ y=\frac{4*1*(-12)-(-1)^2}{4*1}=\frac{-48-1}{4}=\frac{-49}{4} \end{gathered}[/tex]

which means that the solution for the thirst box is:

(1/2, -49/4) (just as in the photo)

Now, the line of symmetry equation of a quadratic function is a vertical line that passes through the vertex, which was calculated to be in the point: (1/2, -49,4).

Because this is a vertical line, it is represented as follows:

[tex]x=\frac{1}{2}[/tex]


Related Questions

A system of equations is shown below:Equation A: 3c = d − 8Equation B: c = 4d + 8Which of the following steps should be performed to eliminate variable d first?Multiply equation A by −4.Multiply equation B by 3.Multiply equation A by 3.Multiply equation B by 4.

Answers

We have the following: system of equations:

A: 3c=d-8

B: c=4d+8

To eliminate variable d first, if we want to use elimination method, we need to have variable d in both equations with the same coefficient but with different signs.

As in equation B, the coefficient of d is 4, then we need to have in equation A a coefficient of -4 for variable d.

Then the answer is we need to multiply equation A by -4.

OA.y> -22² +10z - 8OB. y<-2x² +102-8OC. y2-22² +10r - 8OD. y ≤-22² +10z - 8

Answers

Solution:

Using a graph plotter,

The correct answer that satisfies the graph is OPTION C.

A rectangle is graphed on a coordinate plane and then reflected across the y-axis. If a vertex of the rectangle was at (x, y), which ordered pair represents the corresponding vertex of the new rectangle after the transformation? F (y, x) G (-x, -y) H (-x, y) J (x, y)

Answers

Reflection across y-axisInitial explanation

Let's say that the vertex is the following red point:

Then, its reflection across y-axis would be the blue point:

If we observe the coordinates, we will have that:

(5, 3) is transformed into (-5, 3). This is going to happen no matter the coordinate:

A training field is formed by joining a rectangle and two semicircles, as shown below. The rectangle is 96 m long and 64 m wide. Find the area of the training field. Use the value 3.14 for n, and do not round your answer. Be sure to include the correct unit in your answer.

Answers

To find:

The area of the training field.

Solution:

The training field is made of two semicircles and a rectangle.

The length and width of the rectangle is 96 m and 64 m. So, the area of the rectangle is:

[tex]\begin{gathered} A=l\times w \\ =96\times64 \\ =6144\text{ m}^2 \end{gathered}[/tex]

The diameter of the semicircle is 64 m. SO, the radius of the semicircle is 32 m.

The area of two semicircles is:

[tex]\begin{gathered} A=2\times\frac{1}{2}\pi r^2 \\ =3.14\times(32)^2 \\ =3.14\times1024 \\ =3215.36 \end{gathered}[/tex]

So, the area of the training field is:

[tex]\begin{gathered} A=6144+3215.36 \\ =9359.36 \end{gathered}[/tex]

Thus, the area of the training field is 9359.36 m^2.

Suppose you have $14,000 to invest Which of the two rates would yield the larger amount in 2 years 6% compounded monthly or 5.88% compounded continuously?

Answers

We were given a principal to invest ($14,000) in a timespan of 2 years, and we need to choose between applying it on an account that is compounded montlhy at a rate of 6%, and one that is compounded continuously at a rate of 5.88%. To solve this problem, we need to calculate the final amount on both situations, and compare them.

The expression used to calculate the amount compounded monthly is shown below:

[tex]A=P(1+\frac{r}{12})^{12\cdot t}[/tex]

Where A is the final amount, P is the invested principal, r is the interest rate and t is the elapsed time.

The expression used to calculate the amount compounded continuously is shown below:

[tex]A=P\cdot e^{t\cdot r}[/tex]

Where A is the final amount, P is the invested principal, r is the interest rate, t is the elapsed time, and "e" is the euler's number.

With the two expressions we can calculated the final amount on both situations, this is done below:

[tex]\begin{gathered} A_1=14000\cdot(1+\frac{0.06}{12})^{12\cdot2} \\ A_1=14000\cdot(1+0.005)^{24} \\ A_1=14000\cdot(1.005)^{24} \\ A_1=14000\cdot1.127159 \\ A_1=15780.237 \end{gathered}[/tex][tex]\begin{gathered} A_2=14000\cdot e^{0.0588\cdot2} \\ A_2=14000\cdot e^{0.1176} \\ A_2=14000\cdot1.124794 \\ A_2=15747.12 \end{gathered}[/tex]

The first account, that is compounded monthly yields a return of $15780.24, while the second one that is compounded continuously yields a return of $15747.12, therefore the first account is the one that yield the larger amount in 2 years.

Find the distance between the parallel lines. If necessary, round your answer to the nearest tenths.

Answers

The distance between the parallel lines is [tex]\frac{3}{5}}[/tex].

The given parallel lines are

[tex]y= $-$3x+4\\y= $-$3x+1[/tex]

We have to find the distance between the given parallel lines.

The formula is used to solve the distance between two parallel lines [tex]ax+by+c_{1}=0[/tex] and [tex]ax+by+c_{2}=0[/tex] is

[tex]d=|c_{2} $-$c_{1}|\frac{1}{\sqrt{a^{2}+b^{2}}}[/tex]

The first given line is [tex]y= $-$3x+4[/tex]

We can write that line as [tex]3x$-$y $-$4=0[/tex]

The second given line is [tex]y= $-$3x+1[/tex]

We can write that line as [tex]3x$-$y $-$1=0[/tex]

Comparing the both given parallel lines with the standard equation of line.

After comparing we get

[tex]a=3, b= $-$1, c_{1}= $-$4, c_{2}= $-$1[/tex]

Putting the value in the formula

[tex]d=|(-1) -(-4)|\frac{1}{\sqrt{(3)^{2}+(-4)^{2}}}\\d=|-1+4|\frac{1}{\sqrt{9+16}}\\d=|3|\frac{1}{\sqrt{25}}\\d=\frac{3}{5}}[/tex]

Hence, the distance between the parallel lines is [tex]\frac{3}{5}}[/tex].

To learn more about distance between two parallel lines here link

https://brainly.com/question/10590529

#SPJ1

Move the sliders h and k so that the graph of y = r2 gets shifted up 3 units and to the right 2 units. Then type the new function, f(t) in the answer box 3 2 1 4. بنا -2 0 1 2 3 f(x) -1 h = 0.00 -2 K = 0.00 о Don't forget to shift the graph. Using function notation, i.e. f(x) = , enter the function that results from the transformation.

Answers

Given the graph of the function:

[tex]y=x^2[/tex]

The graph will be shifted 3 units and to the right 2 units

So, the new vertex will be the point ( 2, 3 )

The new function will be:

[tex]f(x)=(x-2)^2+3[/tex]

So, we will adjust the slider on the following values:

[tex]\begin{gathered} h=2 \\ k=3 \end{gathered}[/tex]

I need help with this problem it says to find the area of each shaded sector and round to the hundredth place

Answers

Answer:

1330.81 square feet

Explanation:

In the circle, there are two unshaded sectors with central angles 26° and 90°.

The sum of the central angles = 360°.

Therefore, the sum of the central angle of the shaded sectors will be:

[tex]360\degree-(26\degree+90\degree)=244\degree[/tex]

The area of a sector is calculated using the formula:

[tex]A=\frac{\theta}{360\degree}\times\pi r^2\text{ where }\begin{cases}Central\; Angle,\theta=244\degree \\ Radius,r,HK=25ft\end{cases}[/tex]

Substitute the values into the formula:

[tex]\begin{gathered} A=\frac{244}{360}\times\pi\times25^2 \\ =1330.8136 \\ \approx1330.81\; ft^2 \end{gathered}[/tex]

The area of the shaded sector is 1330.81 square feet (rounded to the hundredth place).

A major record label has seen its annual profit decrease in recent years. In 2011, the label's profit was $128 million. By 2015, the label's profit had decreased by 30%.What was the record label company's profit in 2015? million dollars   Suppose the record label wants to increase its profit to $128 million by 2017. By what percent must the label's profit increase from its 2015 value to reach $128 million within the next two years? %

Answers

the company's profit in 2015 was $89,600,000 (89.6 million dollars)

43%

Explanation:

Profit in 2011 = $128 million

Profit in 2015 decreased by 30%

% decrease = (old price - new price)/old price

old price = Profit in 2011 , new price = Profit in 2015

30% = (128,000,000 - new price)/128000000

[tex]\begin{gathered} 30percent=\text{ }\frac{128,000,000 -newprice}{128000000} \\ 0.30\text{ = }\frac{128,000,000-newprice}{128000000} \\ \text{cross multiply:} \\ 0.3(128,000,000)\text{ = }128,000,000-newprice \end{gathered}[/tex][tex]\begin{gathered} 38400000\text{ = }128,000,000-newprice \\ \text{subtract }38400000\text{ from both sides:} \\ 38400000-\text{ }38400000\text{ = }128,000,000-38400000-newprice \\ \text{0 = 89600000 }-newprice \\ newprice\text{ = 89600000 } \end{gathered}[/tex]

Hence, the company's profit in 2015 was $89,600,000 (89.6 million dollars)

Percentage increase = (new price - old price)/old price

new price = 128million dollars , old price = 89.6 million dollars

% increase = [(128 - 89.6)in millions/(89.6) in millions] × 100

% increase = 38.4/89.6 × 100

% increase = 0.43 × 100

% increase = 43%

Hence, the label's profit must increase by 43% from its 2015 value to reach $128 million within the next two years

Solve the problem. Use 3.14 as the approximate value of pie

Answers

The volume of a cylinder is calculated using the formula:

[tex]V=\pi r^2h[/tex]

where r is the radius of the cylinder and h is the height.

From the question, we have the following parameters:

[tex]\begin{gathered} diameter=4.8 \\ \therefore \\ r=\frac{4.8}{2}=2.4 \\ and \\ h=6.66 \end{gathered}[/tex]

Therefore, we c n calculae tehe volume of a cylinder to be:

[tex]\begin{gathered} V=3.14\times2.4^2\times6.66 \\ V=120.455424 \end{gathered}[/tex]

For four cylinders, the combined volume will be:

[tex]\begin{gathered} V=120.455424\times4 \\ V=481.821696 \end{gathered}[/tex]

The volume i 481 .82 cubic inches.

Find 8 3/4 ÷ 1 2/7. Write the answer in simplest form.

Answers

Problem: Find 8 3/4 ÷ 1 2/7. Write the answer in the simplest form.​

Solution:

[tex](8+\frac{3}{4}\text{ )}\div(1\text{ + }\frac{2}{3})[/tex]

this is equivalent to:

[tex](\frac{32+3}{4}\text{ )}\div(\text{ }\frac{3+2}{3})\text{ = }(\frac{35}{4}\text{ )}\div(\text{ }\frac{5}{3})\text{ }[/tex]

Now, we do cross multiplication:

[tex]=(\frac{35}{4}\text{ )}\div(\text{ }\frac{5}{3})=\frac{35\text{ x 3}}{5\text{ x 4}}\text{ =}\frac{105}{20}[/tex]

then, the correct answer would be:

[tex]=\frac{105}{20}[/tex]

a man pushes a car with a force of 127.5n along a straight horizontal road.he manages to increase the speed of the car from 1 m/s to 2.8 m/s in 12 seconds. find the mass of the car. figure out acceleration first.

Answers

In order to determine the mass of the car, you first calculate the acceleration of the car, by using the following formula:

[tex]a=\frac{v_2-v_1}{\Delta t}[/tex]

where:

v2: final speed of the car = 2.8 m/s

v1: initial speed of the car = 1 m/s

Δt: time interval = 12 s

You replace the previoues values into th formula for the acceleration:

[tex]a=\frac{2.8m/s-1.0m/s}{12s}=0.15\frac{m}{s^2}[/tex]

Next, you the Newton's second law to find the mass of the car. You proceed as follow;

[tex]F=ma[/tex]

where:

m: mass of the car = ?

a: acceleration of the car = 0.15m/s²

F: force exerted on the car by the man = 127.5N

You solve for m in the formula for F, and you replace the values of the other parameters to obtain m, just as follow:

[tex]m=\frac{F}{a}=\frac{127.5N}{0.15m/s^2}=850\operatorname{kg}[/tex]

Hence, the mass of the car is 850kg

what property is used to solve this?

4x-3

x=2

4(2)-3

Answers

commutative property

write a quadratic fuction f whose zeros are -3 and -13

Answers

The zeros of a quadratic function are the points where the graph cuts the x axis.

If one zero is - 3, it means that

x = - 3

x + 3 = 0

Thus, one of the factors is (x + 3)

If another zero is - 13, it means that

x = - 13

x + 13 = 0

Thus, one of the factors is (x + 13)

Thus, the quadratic function would be

(x + 3)(x + 13)

We would open the brackets by multiplyingeach term inside one bracket by each term inside the other. Thus, we have

x * x + x * 13 + 3 * x + 3 * 13

x^2 + 13x + 3x + 39

x^2 + 16x + 39

Thus, the quadratic function is

f(x) = x^2 + 16x + 39

27–34: Describing Distributions. Consider the following distributions.-How many peaks would you expect the distribution to have? Explain.-Make a sketch of the distribution.-Would you expect the distribution to be symmetric, left-skewed, or right-skewed? Explain.-Would you expect the variation of the distribution to be small, moderate, or large? Explain.#29The annual snowfall amounts in 50 randomly selected American cities

Answers

Answer:

Step-by-step explanation:

Identify the constant of variation. 8y-7x=0

Answers

A direct variation between two variables "x" and "y" is given by the following formula:

y = kx

We can rewrite the given expression 8y-7x=0 to get an equation of the form y = kx like this:

8y - 7x = 0

8y - 7x + 7x = 0 + 7x

8y = 7x

8y/8 = 7x/8

y = 7/8x

The number that is being multiplied by x should be the constant of variation k, then in this case, the constant of variation equals 7/8

why are whole numbers rational numbers?

Answers

Answer:

Step-by-step explanation:

A whole number can be written as a fraction that has a denominator of 1. So, the whole numbers 18, 3, and 234 can be written as the rational numbers 18/1, 3/1, and 234/1.

So, all whole numbers are rational numbers, but not all rational numbers are whole numbers.

What is the slope of a line that is perpendicular to the line whose equation is 3x+2y=6?A. −3/2B. −2/3C. 3/2D. 2/3

Answers

We would begin by determining the slope of the line given;

[tex]3x+2y=6[/tex]

To determine the slope, we would have to express the equation of the line in slope-intercept form as follows;

[tex]y=mx+b[/tex]

Therefore, we need to make y the subject of the equation as shown below;

[tex]\begin{gathered} 3x+2y=6 \\ \text{Subtract 3x from both sides of the equation} \\ 2y=6-3x \\ \text{Divide both sides by 2 } \\ \frac{2y}{2}=\frac{6-3x}{2} \\ y=\frac{6}{2}-\frac{3x}{2} \\ y=3-\frac{3}{2}x \end{gathered}[/tex]

The equation in slope-intercept form appears as shown above. Note that the slope is given as the coefficient of x.

Note alo that the slope of a line perpendicular to this one would be a "negative inverse" of the one given.

If the slope of this line is

[tex]-\frac{3}{2}[/tex]

Then, the inverse would be

[tex]-\frac{2}{3}[/tex]

The negative of the inverse therefore is;

[tex]\begin{gathered} (-1)\times-\frac{2}{3} \\ =\frac{2}{3} \end{gathered}[/tex]

The answer therefore is option D

Determine the z-intercepts of the parabola whose graph is given below.

Answers

The x-intercepts are the points where a curve intercepts the x-axis.

From the picture of the problem, we see that the curve intercepts the x-axis in two points:

Answer

The x-intercepts are at:

• x = -6 at (-6,0)

,

• x = -2 at (-2,0)

Find the maximum and minimum values of the function g(theta) = 2theta - 4sin(theta) on the interval Big[0, pi 2 Bigg\

Answers

Hello there. To solve this question, we have to remember some properties about polar curves and determining maximum and minimum values.

In this case, we have the function in terms of the angle θ:

[tex]g(\theta)=2\theta-4\sin(\theta)[/tex]

We want to determine its minimum and maximum values on the closed interval:

[tex]\left[0,\,\dfrac{\pi}{2}\right][/tex]

We graph the function as follows:

Notice on the interval, it has a maximum value of 0.

We can determine its minimum value using derivatives, as follows:

[tex]g^{\prime}(\theta)=2-4\cos(\theta)[/tex]

Setting it equal to zero, we obtain

[tex]\begin{gathered} 2-4\cos(\theta)=0 \\ \Rightarrow\cos(\theta)=\dfrac{1}{2} \\ \\ \Rightarrow\theta=\dfrac{\pi}{3} \end{gathered}[/tex]

Taking its second derivative, we obtain

[tex]g^{\prime}^{\prime}(\theta)=4\sin(\theta)[/tex]

And notice that when calculating it on this point, we get

[tex]g^{\prime}^{\prime}\left(\dfrac{\pi}{3}\right)=4\sin\left(\dfrac{\pi}{3}\right)=2\sqrt{3}[/tex]

A positive value, hence it is a minimum point of the function.

Its minimum value is then given by

[tex]g\left(\dfrac{\pi}{3}\right)=2\cdot\dfrac{\pi}{3}-4\sin\left(\dfrac{\pi}{3}\right)=\dfrac{2\pi}{3}-2\sqrt{3}[/tex]

Of course we cannot determine that 0 is a maximum value of this function using derivatives because it is a local maxima on a certain interval, and derivatives can only gives us this value when the slope of the tangent line is equal to zero.

f(x) = (x ^ 2 + 2x + 7) ^ 3 then

Answers

Answer

[tex]f^{\prime}(x)=6(x+1)(x^{2}+2x+7)^{2}[/tex][tex]f^{\prime}(1)=1200[/tex]

Explanation

Given

[tex]f\mleft(x\mright)=(x^2+2x+7)^3[/tex]

To find the derivative, we have to apply the chain rule:

[tex][u(x)^n]^{\prime}=n\cdot u(x)^{n-1}\cdot u^{\prime}(x)[/tex]

Considering that in our case,

[tex]u(x)=x^2+2x+7[/tex][tex]u^{\prime}(x)=2x+2+0[/tex]

and n = 3, then:

[tex]=3\cdot(x^2+2x+7)^{3-1}\cdot(2x+2)[/tex]

Simplifying:

[tex]f^{\prime}(x)=3\cdot2(x+1)(x^2+2x+7)^2[/tex][tex]f^{\prime}(x)=6(x+1)(x^2+2x+7)^2[/tex]

Finally, we have to replace 1 in each x in f'(x) to find f'(1):

[tex]f^{\prime}(1)=6((1)+1)((1)^2+2(1)+7)^2[/tex][tex]f^{\prime}(1)=6(1+1)(1+2+7)^2[/tex][tex]f^{\prime}(1)=6(2)(10)^2[/tex][tex]f^{\prime}(1)=6(2)(100)[/tex][tex]f^{\prime}(1)=12(100)[/tex][tex]f^{\prime}(1)=1200[/tex]

timmy stated that the product of 3/3 and 12 is greater than the product of 3/2 and 12. is timmy correct?

Answers

[tex]\begin{gathered} \frac{3}{3}\times12=1\times12=12 \\ \frac{3}{2}\times12=3\times6=18 \end{gathered}[/tex]

Hence the product of 3/3 and 12 is not greater than the product of 3/2 and 12.

So timmy is not correct

Please someone can help me please #1

Answers

Complete the  following Division

Quotient of 96, 55, 84 and 63 is 12, 11, 14 and 21 respectively

What is Division?

One of the four fundamental arithmetic operations, or how numbers are combined to create new numbers, is division. The other operations are multiplication, addition, and subtraction.

1) 96

Divisor = 8

96 / 8

= 12

Quotient = 12

2) 55

Divisor = 5

55 / 5

= 11

Quotient = 11

3) 84

Divisor = 6

84 / 6

= 14

Quotient = 14

4) 63

Divisor = 3

63 / 3

= 21

Quotient = 21

Hence , Quotient of 96, 55, 84 and 63 is 12, 11, 14 and 21 respectively

To learn more about Division click on the link

https://brainly.com/question/25289437

#SPJ13

Finding Angles with JustificationIn the diagram below BC = EC and m

Answers

Answer:

Angle Reason

m∠ECD = 140 Given

m∠ECB = 40 Supplementary angles

m∠EBC = 70 Isosceles triangle

m∠ABE = 110 Supplementary angles

Explanation:

Angle ECB and CED are supplementary because they form a straight line and their sum is 180 degrees. So, we can calculate the measure of ∠ECB as

m∠ECB = 180 - 140

m∠ECB = 40

Then, the interior sum of the angles of a triangle is equal to 180 degrees, so

m∠ECB + m∠EBC + m∠BEC = 180

40 + m∠EBC + m∠BEC = 180

However, m∠EBC = m∠BEC because triangle ABC is an isosceles triangle where 2 sides have the same length BC and EC. So, we can find m∠EBC as follows

40 + m∠EBC + m∠EBC = 180

40 + 2m∠EBC = 180

40 + 2m∠EBC - 40 = 180 - 40

2m∠EBC = 140

m∠EBC = 140/2

m∠EBC = 70

Then, the measure of ∠ABE is equal to

∠ABE = 180 - m∠EBC

∠ABE = 180 - 70

∠ABE = 110

Therefore, we can answer it as follows

Angle Reason

m∠ECD = 140 Given

m∠ECB = 40 Supplementary angles

m∠EBC = 70 Isosceles triangle

m∠ABE = 110 Supplementary angles

help meeeeeeeeee pleaseee !!!!!

Answers

The composition of the function, (g o h)(0) = 0.

How to Find the Composition of a Function?

To find the composition of a function, first, find the value of the inner function by plugging in the given value of x. The output of the inner function would now be used as the input to evaluate the outer function.

We are given the following:

g(x) = 5x

h(x) = √x

To find the composition of the function, (g o h)(0), first, find h(0). To find h(0), substitute x = 0 into the inner function, h(x) = √x:

h(0) = √0

h(0) = 0

Find (g o h)(0) by substituting x = 0 into g(x) = 5x:

(g o h)(0) = 5(0)

(g o h)(0) = 0

Learn more about composition of function on:

https://brainly.com/question/10687170

#SPJ1

Suppose that at age 25, you decide to save for retirement by depositing $95 at the end of every month in an IRA that pays 6.25% compounded monthly. How much will you have from the IRA when you retire at age 65? Find the interest.

Answers

1. At age 65 when you retire, you have (future value) $202,531.69 from the IRA.

2. The total interest earned on the monthly investment of $95 at 6.25% for 40 years is $156,931.69.

How is the future value determined?

The future value, which represents the compounded value of the monthly investments, can be computed using the FV formula or an online finance calculator as follows:

Number of years = 40 (65 - 25)

N (# of periods) = 480 months (40 x 12)

I/Y (Interest per year) = 6.25%

PV (Present Value) = $0

PMT (Periodic Payment) = $95

Results:

Future Value (FV) = $202,531.69

Sum of all periodic payments = $45,600 ($95 x 480 months)

Total Interest = $156,931.69

Thus, the future value of the monthly investment is $202,531.69 with an interest of $156,931.69.

Learn more about future value at https://brainly.com/question/24703884

#SPJ1

Car Survey In a survey of 3,100 people who owned a certain type of car, 1,550 said they would buy that type of car again.
What percent of the people surveyed were satisfied with the car?
% of the people surveyed were satisfied with the car.
(Type a whole number.)

Answers

The percentage of people satisfied with car is 50.


What is percentage?
A number or ratio which can be expressed as a fraction of 100 is referred to as a percentage in mathematics. If we need to calculate a percentage of a number, we should divide it by its entirety and then multiply it by 100. The percentage therefore refers to a part per hundred. Per 100 is what the word percent means. The letter "%" stands for it. There is no dimension to percentages. As a result, it is known as a dimensionless number. When we say a number is 50% of something, we mean that it is 50% of everything. As in 0.6%, 0.25%, etc., percentages can also be expressed as decimals or fractions. The grades earned in any subject have been calculated in terms of percentages in academics. Ram, for instance, scored 78% on his exam.

To find the percentage We divide 1550 by 3100 and then multiply by 100

We get

[tex]\frac{1550}{3100}*100\\=50[/tex]

Hence the percentage of people satisfied with the car is 50%

To learn more about percentage please refer

https://brainly.com/question/24304697

#SPJ13

Cynthia wants to buy a rug for a room that is 18ft wide and 28ft long. She wants to leave a uniform strip of floor around the rug. She can afford to buy 264 square feet of carpeting. What dimensions should the rug have ?

Answers

SOLUTION

Let us use a diagram to illustrate the information, we have

Now, from the diagram, let the length of the uniform strip of floor around the rug be x, So, this means the length and width of the rug is

[tex]\begin{gathered} \text{length = 2}8-x-x=28-2x \\ \text{width = }18-x-x=18-2x \end{gathered}[/tex]

Now, since she can afford to buy a rug of 264 square feet for carpeting, this means that the area of the rug is 264, hence we have that

[tex]\begin{gathered} \text{area of rug = (2}8-2x)\times(18-2x) \\ 264=\text{(2}8-2x)(18-2x) \\ \text{(2}8-2x)(18-2x)=264 \end{gathered}[/tex]

Solving for x, we have

[tex]\begin{gathered} \text{(2}8-2x)(18-2x)=264 \\ 504-56x-36x+4x^2=264 \\ 504-92x+4x^2=264 \\ 4x^2-92x+504-264=0 \\ 4x^2-92x+240=0 \end{gathered}[/tex]

Dividing through by 4 we have

[tex]\begin{gathered} x^2-23x+60=0 \\ x^2-20x-3x+60=0 \\ x(x-20)-3(x-20)=0 \\ (x-3)(x-20)=0 \\ x=3\text{ or 20} \end{gathered}[/tex]

So from our calculation, we go for x = 3, because 20 is large look at this

[tex]\begin{gathered} \text{From the length which is (2}8-2x) \\ 28-2(20) \\ =28-40=-12 \end{gathered}[/tex]

length cannot be negative, so we go for x = 3.

Hence the dimensions of the rug becomes

[tex]\begin{gathered} \text{(2}8-2x) \\ =28-2(3) \\ =28-6=22 \\ \text{and } \\ 18-2x \\ 18-2(3) \\ 18-6=12 \end{gathered}[/tex]

So the dimension of the rug should be 22 x 12 feet

Curt and melanie are mixing blue and yellow paint to make seafoam green paint. Use the percent equation to find how much yellowp they should use.

Answers

To solve the exercise you can use the rule of three, like this

[tex]\begin{gathered} 1.5\text{ quarts}\rightarrow100\text{ \% }\Rightarrow\text{ Green paint} \\ x\text{ quarts}\rightarrow30\text{ \%}\Rightarrow\text{ Yellow paint} \end{gathered}[/tex][tex]\begin{gathered} x=\frac{30\text{ \% }\ast\text{ 1.5 quarts}}{100\text{ \%}} \\ x=0.45\text{ quarts} \end{gathered}[/tex]

Therefore, Curt and Melanie should use 0.45 quarts of yellow paint to make seafoam green paint.

I was wondering if you could help me with this problem. I am not sure where to start solving it. Thank you.

Answers

As shown at the graph, we need to find x and y

The angles (x+1) and (2y+1) are vertical

so, x + 1 = 2y + 1

so,

x = 2y eq.(1)

And the sum of the angles (x+1) , (3x + 4y) and (71 - 3y) are 180

So,

(x+1) + (3x + 4y) + (71-3y) = 180

x + 1 + 3x + 4y + 71 - 3y = 180

4x + y = 180 - 1 - 71

4x + y = 108

Substitute with x from eq (1) with 2y

4 * 2y + y = 108

8y + y = 108

9y = 108

y = 108/9 = 12

x = 2y = 2 * 12 = 24

So, x = 24 and y = 12

Other Questions
Complete the steps to find the value of x Gary has read 30 pages of his book. Each day he wants to read 15 pages until he finishes the book which has a total of 180 pages. Write an equation to represent the situation 1. Which scatter plot could have a trend line whose equation is y - 3x + 10 (A) 60 60 40 40 20 20 0 y M 10 20 0 10 20 D . 12 60 8 40 4 29 0 10 220 0 10 10 20 A sporting goods store charges $22.00 for a football before tax. The store holds a sale that marks all prices down by 20% before tax. If the sales tax is 5%, what is the cost of the discounted football after tax? Which graph shows the line y-4-2(x+1)? Element X decays radioactively with a half life of 5 minutes. If there are 860 grams of Element X, how long, to the nearest tenth of a minute, would it take the element to decay to 230 grams? y = a(.5) k Select all the correct answers.Look at the examples in highlighted text. Which shows the woman's problem of self-perceived poverty? (Choose three.) "No...there's nothing so humiliating... Her tastes were simple because... She suffered from the shabbiness of her house.. "Why, darling, I thought you'd be pleased... "Wear flowers," he said. "They're very smart... The diameter of a circle is 6 ft. Find its circumference in terms of \pi. While waiting for the school bus, Michiko records the colors, of all cars passing through an intersection. Thetable shows the results, Estimate the probability that the next car through the intersection will be red. Exgressyour answer as a percent. If necessary, round your anewer to the nearest tenth If 6 is an element in the domain off(x)=82x, what is its corresponding elementin the range Find the distance and the midpoint for each set of points given Which of the following are solutions to the inequality below? Select all that apply. accompanying the bank statement was a credit memorandum for a short-term note collected by the bank for the depositor. this item would be included on the bank reconciliation as a(n) Describe the two different methods shown for writing the complex expression in standard form. Which method do you prefer? Explain The equation of the line that is perpendicular to the line 5x + 6y = 18 and passes through the point (10,7) Natural Logs Propertydo not include any spaces when trying to type in your answer if you have an exponent use ^ Which of the following shows the division problem down below This poem, The Choice, appeared in a book of Dorothy Parkers poems entitled Enough Rope, published in 1925. Most people will recognize that phrase as a shortening of the sentence: Give someone enough rope, and they will hang themselves. A rather dark sentiment, to be sure, but then, as you know, Dorothy Parker had a rather dark sense of humor. For this assignment, write a paragraph in which you support placing this poem in the book of this title, or you argue against the placement of the poem in that book. Be sure to use specific references to the poem to support your position. You may write your response below or type it in a Word document Find the rate of change of each linear function 1. y = x - 7 1P(7,-3); y=x+2Write an equation for the line in point-slope form.(Simplify your answer. Use integers or fractions for any numbers in the equation.)